Articol
Articol
SERIA B
PUBLICAT
, IE LUNARĂ PENTRU TINERET
Introduction
For a subset A ⊆ R denote by A the set of all its real limit points,
i.e. the set of real numbers x for which there exists a sequence (xn )n1 of
numbers from A such that lim xn = x. The set A is called closed if A ⊆ A.
n→∞
For instance, any interval [a, b] is closed.
Whenever dealing with continuous functions, it is natural to consider
the set A , where A is a set that is easy to work with. A well-known and very
important result that we will use throughout this paper is that the intersec-
tion of two closed sets is also closed. Moreover, we will use the infimum and
supremum of a set. It is easy to prove that inf A and sup A belong to A ,
provided that they are real numbers.
Next, we will present the statements of the main results and prove them,
followed by some examples of olympiad problems.
The two main properties
Lemma 1 (bounded version). Consider a nonempty bounded set
M ⊆ R with the following two properties:
i) M is closed;
1) Elev, Liceul Teoretic Internat, ional de Informatică, Bucures, ti.
282 Articole s, i note matematice
Olympiad Problems
Now, we are ready to show how to use these two properties for contest-
type problems. The main trick for each problem is to choose a set for which
one of the two lemmas from above can be applied.
1. (Mihai Piticari, Romanian Mathematical Olympiad 2005, District
Round) Let f : R → R be a continuous function such that for any a, b ∈ R,
with a < b and f (a) = f (b), there exists some c ∈ (a, b) such that f (a) =
f (b) = f (c). Prove that f is monotonic over R.
Solution. Assume that f is not monotonic. Then it is easy to prove
that there exist a < b < c such that
f (a) < f (b) and f (b) > f (c)
or
f (a) > f (b) and f (b) < f (c).
Assume we are in the first case, as the second one can be treated in a
similar way. Choose λ such that
f (b) < λ < max{f (a), f (c)}
Since f is continuous, it follows from the intermediate value theorem that
there exist x, y such that a < x < b < y < c and f (x) = f (y) = λ. Hence, if
we consider the set
M = {t | t ∈ [x, y] and f (t) = λ},
then x, y ∈ M and M is clearly bounded. From the continuity of f it follows
that M is closed. Moreover, from the problem hypothesis, it is clear that for
each u < v from M there exists w ∈ (u, v) such that w ∈ M .
So we can apply Lemma 1, hence M is a closed interval. Since x, y ∈ M
and x < b < y, then b ∈ M , which is a contradiction since f (b) < λ. The
conclusion follows.
2. (Marius Cavachi, Romanian Mathematical Olympiad 2020, District
Round, 2020) Determine the continuous functions f : R → R having the
property that, for all x, y ∈ R, there exists t ∈ (0, 1) such that
f ((1 − t)x + ty) = (1 − t)f (x) + tf (y).
284 Articole s, i note matematice
yf (x)−xf (y)
and n = y−x . Define the set
M = {t ∈ [x, y] | g(t) 0}
and note that x, y ∈ M , since g(x) = g(y) = 0.
Since f is continuous, then g is also continuous. Moreover, it is easy to
check that g has property P, since the affine function is canceled out from
both sides. From the continuity of g, it follows that M is closed. Choose
u < v from M . Since g has property P, then there exists w ∈ (u, v) such
that
(v − u)g(w) (v − w)g(u) + (w − u)g(v).
Since v > u and the right hand side term is nonpositive, then w ∈ M , which
shows that M has property (ii). As M is clearly bounded, we can apply
Lemma 1 to infer that M = [x, y].
So, for every z ∈ (x, y), f (z) mz + n. Substituting the values of m, n
in this inequality easily leads to
(y − x)f (z) (y − z)f (x) + (z − x)f (y).
In particular, for z = λx + (1 − λ)y this yields
f (λx + (1 − λ)y) λf (x) + (1 − λ)f (y),
which is a contradiction with the initial assumption on x, y and λ. This
proves that f must be convex on R.
4. (after a problem of Dan Marinescu) A function f : R → R is said
to have property P if for every a, b ∈ R there exists c ∈ (a, b) such that
f (c) ∈ {f (a), f (b)}, where the brackets can also denote a multiset.
a) Give an example of a non-constant function possessing P.
b) If f has P and is continuous, prove that it is constant.
Solution. a) The Dirichlet function
1, if x ∈ Q
f (x) =
0, if x ∈ R \ Q
is not constant and property P follows from the density of both Q and R \ Q
in R.
b) Assume that f is not constant and pick real numbers a < b, such
that f (a) = f (b). Define the set
M = {x ∈ [a, b] | f (x) = f (a) or f (x) = f (b)}.
It is clear that a, b ∈ M . Choose u < v from M . Since f has property P,
then there exists w ∈ (u, v) such that f (w) = f (u) or f (w) = f (v). Since
u, v ∈ M then
f (u), f (v) ∈ {f (a), f (b)} ⇒ f (w) ∈ {f (a), f (b)}
hence w ∈ M .
286 Articole s, i note matematice
References
[1] The Romanian Mathematical Competitions Collection 2004-2020.